Những câu hỏi liên quan
nguyen thi be
Xem chi tiết
Ami Mizuno
20 tháng 7 2021 lúc 15:46

Ami Mizuno
20 tháng 7 2021 lúc 15:52

nguyen thi be
Xem chi tiết
Nguyễn Việt Lâm
19 tháng 7 2021 lúc 10:50

\(V_{SABC}=\dfrac{1}{3}SA.\dfrac{1}{2}AB.AC=\dfrac{a^3\sqrt{3}}{6}\)

\(S_{SAM}=\dfrac{1}{2}S_{SAB}\Rightarrow V_{SAMC}=\dfrac{1}{2}V_{SABC}=\dfrac{a^3\sqrt{3}}{12}\)

Tam giác SAB vuông tại A nên AM là trung tuyến ứng với cạnh huyền

\(\Rightarrow AM=\dfrac{1}{2}SB=\dfrac{1}{2}\sqrt{SA^2+AB^2}=a\)

\(\left\{{}\begin{matrix}SA\perp\left(ABC\right)\Rightarrow SA\perp AC\\AC\perp AB\end{matrix}\right.\) \(\Rightarrow AC\perp\left(SAB\right)\Rightarrow AC\perp AM\) 

Hay tam giác ACM vuông tại M

\(\Rightarrow S_{AMC}=\dfrac{1}{2}AM.AC=\dfrac{a^2}{2}\)

\(\Rightarrow d\left(S;\left(AMC\right)\right)=\dfrac{3V_{SAMC}}{S_{AMC}}=\dfrac{a\sqrt{3}}{2}\)

Pham Trong Bach
Xem chi tiết
Cao Minh Tâm
21 tháng 3 2019 lúc 17:09

Chọn A.

Gắn tọa độ Oxyz, với A(0;0;0), B(1;0;0), D(0;3;0), S(0;0;1)

Khi đó C ( 1 ; 3 ; 0 ) ⇒  Trung điểm M của BC là M ( 1 ; 3 2 ; 0 ) .  

Ta có

SM → = ( 1 ; 3 2 ; - 1 ) , SD →   = ( 0 ; 3 ; - 1 ) ⇒ [ SM →   ; SD → ] = ( 3 2 ; 1 ; 3 ) .  

Suy ra n ⃗ ( SDM ) = ( 3 2 ; 1 ; 3 )  mà n ⃗ ( ABCD ) = n ⃗ ( Oxy ) = ( 0 ; 0 ; 1 ) ,  

ta được

cos ( SDM ^ ) ;   ( ABCD )   =   n → ( SDM ) . n → ( ABCD ) n → ( SDM ) . n → ( ABCD ) = 6 7 .

nguyen thi be
Xem chi tiết
Nguyễn Việt Lâm
6 tháng 5 2021 lúc 21:03

1.

\(SA\perp\left(ABC\right)\Rightarrow AC\) là hình chiếu vuông góc của SA lên (ABC)

\(\Rightarrow\widehat{SCA}\) là góc giữa SC là (ABC)

\(AC=\sqrt{AB^2+BC^2}=a\sqrt{5}\)

\(\Rightarrow tan\widehat{SCA}=\dfrac{SA}{AC}=\dfrac{\sqrt{15}}{5}\Rightarrow\widehat{SCA}\approx37^045'\)

b.

Ta có: \(\left\{{}\begin{matrix}SA\perp\left(ABC\right)\Rightarrow SA\perp BC\\AB\perp BC\end{matrix}\right.\) \(\Rightarrow BC\perp\left(SAB\right)\)

Mà \(\left\{{}\begin{matrix}BC=\left(SBC\right)\cap\left(ABC\right)\\SB=\left(SBC\right)\cap\left(SAB\right)\\AB=\left(ABC\right)\cap\left(SAB\right)\end{matrix}\right.\) \(\Rightarrow\widehat{SBA}\) là góc giữa (SBC) và (ABC)

\(tan\widehat{SBA}=\dfrac{SA}{AB}=\sqrt{3}\Rightarrow\widehat{SBA}=60^0\)

c.

Trong mp (SAB), từ A kẻ \(AH\perp SB\)

Mà \(BC\perp\left(SAB\right)\Rightarrow BC\perp AH\)

\(\Rightarrow AH\perp\left(SBC\right)\Rightarrow AH=d\left(A;\left(SBC\right)\right)\)

Áp dụng hệ thức lượng:

\(\dfrac{1}{AH^2}=\dfrac{1}{SA^2}+\dfrac{1}{AB^2}\Rightarrow AH=\dfrac{SA.AB}{\sqrt{SA^2+AB^2}}=\dfrac{a\sqrt{3}}{2}\)

Nguyễn Việt Lâm
6 tháng 5 2021 lúc 21:14

2.

\(SA\perp\left(ABCD\right)\Rightarrow AB\) là hình chiếu vuông góc của SB lên (ABCD)

\(\Rightarrow\widehat{SBA}\) là góc giữa SB là (ABCD)

\(tan\widehat{SBA}=\dfrac{SA}{AB}=3\Rightarrow\widehat{SBA}\approx71^034'\)

b.

\(\left\{{}\begin{matrix}SA\perp\left(ABCD\right)\Rightarrow SA\perp BC\\AB\perp BC\end{matrix}\right.\) \(\Rightarrow BC\perp\left(SAB\right)\)

Mà \(\left\{{}\begin{matrix}BC=\left(SBC\right)\cap\left(ABCD\right)\\SB=\left(SAB\right)\cap\left(SBC\right)\\AB=\left(SAB\right)\cap\left(ABCD\right)\end{matrix}\right.\)

\(\Rightarrow\widehat{SBA}\) là góc giữa (SBC) và (ABCD) (đã tính ở câu a)

c.

Từ A kẻ \(AH\perp SB\Rightarrow AH\perp\left(SBC\right)\)

\(\Rightarrow AH=d\left(A;\left(SBC\right)\right)\)

\(\dfrac{1}{AH^2}=\dfrac{1}{SA^2}+\dfrac{1}{AB^2}\Rightarrow AH=\dfrac{SA.AB}{\sqrt{SA^2+AB^2}}=\dfrac{3a\sqrt{10}}{10}\)

nguyen thi be
Xem chi tiết
Pham Trong Bach
Xem chi tiết
Cao Minh Tâm
6 tháng 11 2017 lúc 17:18

 phụ nhau nên  D I A ^ = 90 °

Ngọc Hưng
Xem chi tiết
Nguyễn Việt Lâm
24 tháng 3 2022 lúc 11:19

\(\widehat{BAD}=120^0\Rightarrow\widehat{ABC}=60^0\Rightarrow\) các tam giác ABC và ACD là các tam giác đều

\(AH=AC\Rightarrow AH=AC=AB\Rightarrow\Delta HBC\)  vuông tại B

\(\Rightarrow HB\perp BC\Rightarrow HB\perp AD\)

Qua H kẻ đường thẳng \(d\perp\left(ABCD\right)\Rightarrow S\in d\)

Gọi O là giao điểm AC và BD, do góc giữa (SBD) và đáy bằng 60 độ

\(\Rightarrow\widehat{SOH}=60^0\)

\(\Rightarrow SH=OH.tan60^0=\left(AH+AO\right).tan60=\left(a+\dfrac{a}{2}\right).tan60^0=\dfrac{3a\sqrt{3}}{2}\)

\(V_{S.ABCD}=\dfrac{1}{3}SH.S_{ABCD}=\dfrac{1}{3}.\dfrac{3a\sqrt{3}}{2}.2.\dfrac{a^2\sqrt{3}}{4}=\dfrac{3a^3}{4}\)

 

Nguyễn Việt Lâm
24 tháng 3 2022 lúc 11:19

b.

\(SC=\sqrt{SH^2+HC^2}=\sqrt{SH^2+\left(2AC\right)^2}=\dfrac{a\sqrt{43}}{2}\)

\(\Rightarrow M\) là trung điểm SC \(\Rightarrow AM\) là đường trung bình tam giác SHC

\(\Rightarrow\left\{{}\begin{matrix}AM||SH\Rightarrow AM\perp\left(ABCD\right)\\AM=\dfrac{1}{2}SH=\dfrac{3a\sqrt{3}}{4}\end{matrix}\right.\)

\(HD=\sqrt{OD^2+OD^2}=\sqrt{\left(\dfrac{3a}{2}\right)^2+\left(\dfrac{a\sqrt{3}}{2}\right)^2}=a\sqrt{3}\)

\(\Rightarrow SD=\sqrt{SH^2+HD^2}=\dfrac{a\sqrt{39}}{2}\Rightarrow SN=\dfrac{1}{3}SD\Rightarrow ND=\dfrac{2}{3}SD\)

\(\Rightarrow d\left(N;\left(MAD\right)\right)=\dfrac{2}{3}d\left(S;\left(MAD\right)\right)\)

Mà \(SH||\left(MAD\right)\Rightarrow d\left(S;\left(MAD\right)\right)=d\left(H;\left(MAD\right)\right)\)

Gọi E là giao điểm BH và AD, ta có:

\(\left\{{}\begin{matrix}HB\perp AD\left(cmt\right)\\AM\perp\left(ABCD\right)\Rightarrow AM\perp HB\end{matrix}\right.\) \(\Rightarrow HB\perp\left(MAD\right)\)

\(\Rightarrow HE=d\left(H;\left(MAD\right)\right)\)

\(HE=\dfrac{1}{2}HB=\dfrac{1}{2}HD=\dfrac{a\sqrt{3}}{2}\)

\(\Rightarrow d\left(N;\left(MAD\right)\right)=\dfrac{2}{3}HE=\dfrac{a\sqrt{3}}{3}\)

\(\Rightarrow V_{AMND}=\dfrac{1}{3}.d\left(N;\left(MAD\right)\right).\dfrac{1}{2}AM.AD=\dfrac{a^3}{8}\)

Nguyễn Việt Lâm
24 tháng 3 2022 lúc 11:21

undefined

Pham Trong Bach
Xem chi tiết
Cao Minh Tâm
4 tháng 5 2018 lúc 18:18

ĐÁP ÁN: D

nguyen thi be
Xem chi tiết
Akai Haruma
25 tháng 7 2021 lúc 9:48

1.

Gọi $I$ là trung điểm $AB$ thì do tam giác $DAB$ và $CAB$ cân tại $D$ và $C$ nên:

$DI\perp AB; CI\perp AB$

$\Rightarrow (DCI)\perp AB$

$\Rightarrow (DCI)\perp AI$ và $(DCI)\perp BI$

Do đó:

\(V_{ABCD}=V_{DAIC}+V_{DIBC}=\frac{1}{3}AI.S_{DIC}+\frac{1}{3}BI.S_{DIC}\)

\(=\frac{1}{3}S_{DIC}(AI+BI)=\frac{1}{3}S_{DIC}.AB=\frac{x}{3}S_{DIC}\)

\(DI=\sqrt{DA^2-AI^2}=\sqrt{DA^2-(\frac{AB}{2})^2}=\sqrt{12-\frac{x^2}{4}}\)

\(CI=\sqrt{AC^2-AI^2}=\sqrt{AC^2-(\frac{AB}{2})^2}=\sqrt{12-\frac{x^2}{4}}\)

$\Rightarrow DCI$ là tam giác cân tại $I$

Kẻ $IM\perp DC$ thì $M$ là trung điểm $DC$

$IM=\sqrt{DI^2-DM^2}=\sqrt{12-\frac{x^2}{4}-(\sqrt{3})^2}$

$=\sqrt{9-\frac{x^2}{4}}$

\(S_{DIC}=\frac{IM.DC}{2}=\sqrt{9-\frac{x^2}{4}}.2\sqrt{3}:2=\frac{\sqrt{3}.\sqrt{36-x^2}}{2}\)

Vậy: \(V_{ABCD}=\frac{\sqrt{3}}{6}x\sqrt{36-x^2}=\frac{\sqrt{3}}{6}\sqrt{x^2(36-x^2)}\)

\(\leq \frac{\sqrt{3}}{6}.\frac{x^2+36-x^2}{2}=3\sqrt{3}\) theo BĐT Cô-si

Vậy $V_{ABCD}$ max bằng $3\sqrt{3}$ khi $x^2=36-x^2$
$\Leftrightarrow x=3\sqrt{2}$

Akai Haruma
25 tháng 7 2021 lúc 9:51

Hình bài 1

Akai Haruma
25 tháng 7 2021 lúc 9:56

Bài 2:
Kẻ $AT\perp $SB$

Ta có:

$SA\perp (ABCD)\Rightarrow SA\perp BC$

$AB\perp BC$ (do $ABCD$ là hình vuông)

$\Rightarrow (SAB)\perp BC$

$\Rightarrow AT\perp BC$ (vì \(AT\subset (SAB)\) )

Mà: $AT\perp SB$

$\Rightarrow AT\perp (SBC)$

$\Rightarrow AT=d(A, (SBC))=\frac{a\sqrt{2}}{2}$

$\frac{1}{SA^2}+\frac{1}{AB^2}=\frac{1}{AT^2}$ theo hệ thức lượng 

$\Leftrightarrow \frac{1}{SA^2}=\frac{1}{AT^2}-\frac{1}{AB^2}=\frac{2}{a^2}+\frac{1}{a^2}$

$\Rightarrow SA=\frac{\sqrt{3}a}{3}$

$V_{S.ABCD}=\frac{1}{3}.SA.S_{ABCD}=\frac{1}{3}.\frac{a\sqrt{3}}{3}.a^2=\frac{\sqrt{3}}{9}a^3$